Journalist: People whose diets contain a relatively large amount of iron are significantly more likely to develop Par...

rmkrutz@crimson.ua.edu on May 8, 2018

Please Explain

Could you please explain the answer choice A?

Reply
Create a free account to read and take part in forum discussions.

Already have an account? log in

Christopher on May 12, 2018

@rmkrutz@crimson.ua.edu I remember scratching my head at this one when I was prepping for the LSAT, and this is a question that, I feel, is most easily solved through process of elimination.

The journalist's argument comes down to saying that if you limit the amount of iron rich food you eat, you should reduce your chances of developing Parkinson's disease. So let's look at how each answer interacts with the argument.

(A) eliminates an alternate explanation and potential objection to the argument, thus strengthening it. If you were to demonstrate that people with a genetic predisposition to Parkinson's ALL had high-iron diets it would weaken the premise that high iron intake increases the risk of Parkinson's by providing an alternate explanation. Thus by saying that most do NOT have a relatively higher amount of iron in their diet, it is basically saying "all other things being equal, higher iron intake increases the Parkinson's risk," allowing the journalist to come to the conclusion in the question.

(B) compares the amount of iron in vegetables to that in meat and seafood, but the journalist doesn't exclude vegetables from the argument. It lists "meat, seafood, and other food rich in iron" making the comparison between vegetables and meat/seafood irrelevant.

(C) compares the amount of iron needed for different age groups, but this doesn't interact with the argument in any way. Nothing in the premise or conclusion mention age or minimum requirements for iron.

(D) looks at iron absorption rates of various foods. This would make a difference if the argument had to do with eating seafood over meat, but that's not part of this argument. The journalist is pointing to all iron rich food, so this does nothing to the argument.

(E) addresses the eating habits of people over age 50. This doesn't address anything in the conclusion or the premise.

This is a tricky question since the right answer doesn't directly strengthen the argument but rather addresses a weakness. However, the other answers are fairly easy to eliminate if you take each one on its own merit.